LSAT and Law School Admissions Forum

Get expert LSAT preparation and law school admissions advice from PowerScore Test Preparation.

 Administrator
PowerScore Staff
  • PowerScore Staff
  • Posts: 8937
  • Joined: Feb 02, 2011
|
#22919
Complete Question Explanation

Justify the Conclusion-SN. The correct answer choice is (A)

The argument is that since a photograph could be made to misrepresent a scene, it cannot express the whole truth and cannot prove anything.

The argument is highly questionable; however, we are asked to justify the conclusion. Because the stimulus essentially provides conditions, it is likely that the correct choice will be an additional conditional statement that forces the conclusion. Currently, we should notice that the fact that a photograph cannot express the whole truth does not lead to the conclusion that nothing at all could be proved, so a link is needed.

Answer choice (A): This is the correct answer choice. If things that cannot establish the whole truth cannot furnish definitive proof, the conclusion is justified. We could make the conditional diagram:
  • Photograph .......... Not Whole Truth .......... Not Definitive Proof.
Answer choice (B): This choice makes it likely that if the argument is correct, nothing can ever be proved. However, it does not supply the link between the lack of whole truth and the inability to prove, and is incorrect. If you chose this answer, you may have misread either it or the concluding sentence of the stimulus.

Answer choice (C): This answer choice is contrary to the first sentence of the stimulus, and is incorrect.

Answer choice (D): This answer seeks to attack the conclusion rather than to establish it, and is incorrect.

Answer choice (E): This answer choice is actually irrelevant, although a misread would attack the stimulus. In any case, the choice is about establishing truth while taking the photograph, and not the photo itself.
 Nadia0702
  • Posts: 53
  • Joined: Sep 19, 2013
|
#11876
Hello PS,

I solved question 17 Mechanistically by identifying the rogue element "definitively proved" in the conclusion and then narrowing it down to choices A/E. I chose A because E essentially contradicts the second premise. Even though I got the correct answer, I am trying to diagram all conditional statements and use the Justify Formula on all questions to ensure my understanding. On this question, I thoroughly confused myself with whether or not I recognized conditional statements correctly. Here is what I have:

P1. Conditional Statement:
Photograph :arrow: Is True

P2. Conditional Statement??
Photograph :arrow: Cannot Express Whole Truth :arrow: Is False
A :arrow: B

C: Conditional Statement???
Photograph :arrow: Nothing can ever be definitely proved with it
A :arrow: C

Justify formula: I need B :arrow: C??
If something is False, then nothing can ever be can be definitively proved with it. This sort of agrees with answer choice A.

Did I do the above diagramming correct? Or did I create conditional statements where there were none?

Many thanks for the help!
Nadia
User avatar
 KelseyWoods
PowerScore Staff
  • PowerScore Staff
  • Posts: 1079
  • Joined: Jun 26, 2013
|
#11893
Hi Nadia,

Your diagramming is correct. Excellent diagramming and use of the Justify Formula!

Best,
Kelsey
 mpoulson
  • Posts: 148
  • Joined: Mar 25, 2016
|
#23026
Hello,

I wanted help to draw the conditional statements employed in the stimulus of Question 17 and how to arrive logically at Answer A. I thought it went something like this ~Expressing whole truth is the sufficient condition for being false and being false is needs to be sufficient condition for ~ definitive proof. Therefore that would arrive at answer A. Because if something is false (cannot express whole truth) then it cannot be definitive proof. Let me know if I am on the right track or if I missed a step. Thank you.

Respectfully,

Micah
 Robert Carroll
PowerScore Staff
  • PowerScore Staff
  • Posts: 1787
  • Joined: Dec 06, 2013
|
#23071
Micah,

This particular stimulus is a nice situation to illustrate the concept on page 4-28: "Solving Justify Questions Mechanistically." The last sentence expresses the conclusion, and that conclusion says that "nothing can ever be definitely proved." But nothing in the premises said that nothing can ever be definitively proved with a photograph - the premises talked about the lack of expressing the whole truth. Thus the conclusion contains new information. You can immediately expect the correct answer to link the premise information to the "nothing can ever be definitely proved" concept in the conclusion.

You are definitely on the right track. You want something in the premises to be sufficient to show that the conclusion is necessary - that would allow you to justify the conclusion, and thus answer this Justify question. Answer choice (A) makes the necessary link. If the kind of falsity in the premises (inability to express the whole truth) is sufficient to prove there cannot be definitive proof, then the argument's conclusion follows.

Robert Carroll
 Blueballoon5%
  • Posts: 156
  • Joined: Jul 13, 2015
|
#44565
Hi! Is "cannot" a necessary condition indicator with a special rule (like unless, except, until, and without)?
 James Finch
PowerScore Staff
  • PowerScore Staff
  • Posts: 943
  • Joined: Sep 06, 2017
|
#44582
Hi Blueballoon,

No, cannot isn't a conditional indicator at all, but a modifier (adverb) that means that whatever verb/condition won't happen. In this case it serves to negate the "express whole truth" condition and make our diagram look like:

May be shown differently :arrow: express whole truth :arrow: false in a sense.

Hope this helps!
 deck1134
  • Posts: 160
  • Joined: Jun 11, 2018
|
#46866
Hello,

On Conditional Justify Questions, the Forum has used two diagrams to explain what we are looking for.

A----->B
A----->C


Some say we need
B----->C.

Could we also do
NOTB-------NOTA?
User avatar
 Dave Killoran
PowerScore Staff
  • PowerScore Staff
  • Posts: 5925
  • Joined: Mar 25, 2011
|
#46867
Hi Deck,

No, that statement is the contrapositive of the first premise, and so is identical in meaning to that premise. As such, it just repeats part of the argument. Instead, we need a connection between previously unconnected parts.

You rightly mention that B :arrow: C would work, and it's also the case that the contrapositive of that would work:

..... ..... C :arrow: B

If that's the statement you meant, then yes, you are correct :-D

Note that because we make the point that a statement and its contrapositive are identical in meaning, and that the CP is always present for any conditional statement, when we say a certain conditional statement (such as B :arrow: C) works, we usually won't then make the point that the contrapositive also works.

Please let me know if that helps. Thanks!

Get the most out of your LSAT Prep Plus subscription.

Analyze and track your performance with our Testing and Analytics Package.